Emergency Medicine GI/Abdomen

Ace your homework & exams now with Quizwiz!

A disheveled 42-year-old man with no known past medical history presents with mid-epigastric pain which radiates through to his back. He admits to chronic alcoholism and repeated prior episodes of similar pain. The patient states that he has neither seen a physician nor been to an Emergency Department in over 20 years. His exam reveals a tender but non-rigid abdomen. Which of the following is an appropriate management plan? Abdominal CT scan Administration of lactulose orally to alter ammonia reabsorption in the gut Immediate triage to an outpatient alcohol rehabilitation program Withholding opioid pain medication secondary to the likelihood the patient is drug seeking

Correct Answer ( A ) Explanation: Chronic pancreatitis is an ongoing inflammatory process leading to irreversible structural damage and impairment of the exocrine and endocrine pancreatic function. Patients with chronic pancreatitis may present with near-constant and intractable chronic pain. The risk increases with the duration and amount of alcohol consumption. Chronic inflammation can cause local injury resulting in lesions such as pseudoaneurysms, splenic vein thrombosis, pancreatic ascites, or pancreatic fistulas. Pancreatic pseudocyst formation is seen in up to 25% of patients with chronic pancreatitis. Patients with chronic pancreatitis have an increased risk for the development of pancreatic carcinoma. The diagnosis of chronic pancreatitis is often made clinically. The serum levels of amylase and lipase initially are mildly elevated in chronic pancreatitis, but as the disease progresses, these levels normalize. In patients with clinical manifestations of chronic pancreatitis, including pain, malabsorption, and diabetes, the diagnosis is made by CT scan. CT shows dilated intrahepatic ducts, microcalcifications, pseudocysts, or other complications. In the ED setting, patients with a known history of chronic pancreatitis do not need imaging except when the underlying cause of the pain is in question or if the pain is prolonged, significantly increased, or unresponsive to treatment. Lactulose (B) is administered to patients with hepatic encephalopathy, not pancreatitis. Both are related to chronic alcohol consumption. The patient requires acute management and outpatient alcohol rehabilitation (C) can take place after the patient is stabilized and his pain is controlled. Patients with chronic pancreatitis often experience intractable pain withholding pain medication (D) is inappropriate.

Which of the following is true regarding elderly patients with abdominal pain? More likely to need an emergent surgical procedure More likely to present with an elevated white blood cell count More likely to present with fever More likely to present with peritoneal signs

Correct Answer ( A ) Explanation: Older patients with abdominal pain are more likely to present atypically and more likely to present with life-threatening causes requiring emergent surgery. Abdominal pain can be a diagnostic challenge in all patients due to the variability in presentation of common intraabdominal pathologies. Often, an exact etiology is not found on initial evaluation. However, the likelihood of having a benign cause decreases with increasing age. Older patients with abdominal pain are more likely to harbor serious pathology but will often present atypically and progress rapidly. The decrease in diagnostic accuracy and increased rate of severe disease results in an increased mortality for older patients. In those older patients where specific causes are found, they are more likely to require a surgical procedure. Fever (C), elevated WBC counts (B) and peritoneal pain (D) are less common in older patients in comparison to younger patients.

A 54-year-old man with cirrhosis presents for evaluation of abdominal pain. The pain is diffuse throughout the abdomen and associated with subjective fever at home. He has no vomiting, diarrhea or change in mental status. His vital signs are T 100.6°F, HR 102, BP 140/88, RR 12, and oxygen saturation of 100% on room air. Bedside ultrasound demonstrates ascites. Which of the following is an indication for intravenous antibiotics? Ascitic fluid neutrophil count of 300 cells/mcL Ascitic fluid pH of 7.35 AST of 340 U/L Peripheral white blood cell count of 15,000 cells/mcL

Correct Answer ( A ) Explanation: The patient's presentation is concerning for spontaneous bacterial peritonitis (SBP), an acute bacterial infection in the ascitic fluid of patients with ascites in the setting of liver disease. Most commonly, gram negative enteric organisms are responsible for the infection and the treatment of choice is an intravenous third generation cephalosporin. Diagnosis is made based on an ascitic neutrophil count > 250 cells. Other test results have been correlated with SBP, but treatment is guided based on the neutrophil count of the ascitic fluid. An ascitic pH of 7.35 (B) is nonspecific for this illness. A pH less than 7.34 or a gradient of more than 0.10 between the arterial and ascitic pH may be an earlier indicator of early SBP. An AST of 340 (C) is not predictive of SBP. Patients with cirrhosis will often have elevated transaminases, and particularly alcoholic patients will have an AST higher than ALT. The peripheral white blood cell count of 15,000 (D) may indicate the presence of infection, but is not specific and does not mandate the initiation of intravenous antibiotic therapy.

A 54-year-old man presents complaining of epigastric pain that started several hours ago. The pain is moderate and sharp but does not radiate. He has a history of hypertension and exploratory laparotomy 20 years ago. His vital signs on presentation are T 37°C, HR 95, and BP 136/80. His exam reveals a firm, mildly tender protruding mass in the epigastric region with no overlying skin changes. Which of the following is the next best step? Apply gentle steady pressure to the mass Obtain computed tomography scan of the abdomen Perform bedside abdominal aortic ultrasound Perform incision and drainage of the mass

Correct Answer ( A ) Explanation: This patient is presenting with a ventral hernia through the incision site from his exploratory laparotomy. Incisional hernias account for up to 20% of all abdominal wall hernias. They are often the result of excess wall tension or inadequate wound healing. Risk factors for the development of incisional hernias include obesity, age, wound infection, and certain medical conditions that increase the intraabdominal pressure. All hernias fall into one of three categories: reducible, incarcerated, or strangulated. Reducible hernias are soft and easy to replace through the hernia defect. Incarcerated hernias are firm, often painful, and non-reducible by direct manual pressure. Strangulation occurs as a consequence of incarceration and results in impaired blood flow, leading to ischemia, necrosis, and obstruction. Skin changes overlying the hernia site may be seen, and patients are often toxic in appearance. A strangulated hernia is a true surgical emergency. Abdominal aortic aneurysm (C) should be considered in this patient. However, this condition generally presents as a pulsatile mass in patients with chronic hypertension. An incision and drainage (D) is appropriate for an abscess. This mass is firm, whereas an abscess is fluctuant. To reduce the potential for ischemia and necrosis, there should be no delay at reduction (B).

A 40-year-old man presents with rectal pain and fever. Physical examination reveals an indurated left buttock with fluctuance tracking towards the anus. Digital examination reveals tenderness and fluctuance. What is the next best step in management? CT scan and surgical consultation Incision and drainage in the emergency department Intravenous antibiotics Oral antibiotics and outpatient follow up

Correct Answer ( A ) Explanation: This patient presents with signs and symptoms concerning for a deep anorectal abscess and requires a CT scan for diagnosis and surgical consultation for operative management. Some anorectal abscesses can be treated with simple incision and drainage in the Emergency Department (perianal). However, the deeper abscesses cannot be adequately drained without anesthesia. Ischiorectal, intersphincteric, supralevator and postanal abscesses typically require more invasive management. These abscesses often present with rectal pain and may have associated fever. External examination may not reveal the full extent of the abscess, but patients often have severe tenderness on rectal examination. In intersphinchteric and supralevator abscesses, rectal examination may reveal fluctuance or a tender mass. CT scan of the pelvis can delineate the extent of disease and aid in guiding management.

A 2-year-old boy is brought to the ED with a suspected battery ingestion. One hour prior to presentation, the child was playing with a remote control and the parents noticed he had a brief choking episode. Subsequently, the disc battery that powered the device was missing. The child is currently asymptomatic with normal vital signs and an unremarkable physical exam. An x-ray is performed that shows the battery lodged in the midesophagus. What is the most appropriate next step in the management of this patient? Attempt blind removal with a catheter Immediate endoscopy and removal Induce vomiting with ipecac Laxatives to promote passage of the battery No intervention is necessary at this time

Correct Answer ( B ) Explanation: A battery lodged in the esophagus can cause significant corrosive injury in as little as two hours. This is a medical emergency that requires immediate intervention. Endoscopy is the intervention of choice unless there is suspicion of esophageal perforation or significant bleeding. Endoscopy allows for direct visualization of the mucosa, can determine the severity of injury, and provides important prognostic information. Batteries cause injury because electrical current generated hydrolyzes body fluids and forms hydroxide ions, which cause alkaline burns. Leakage of alkaline battery contents and pressure necrosis are minor mechanisms of injury. Complications of battery ingestions include esophageal ulceration, perforation, stricture formation, fistula formation, and vocal cord paralysis. The most common cause of death is aortoesophageal fistula formation, which can cause delayed hemorrhage. Batteries visualized in the stomach or beyond will generally pass without complication and rarely require intervention unless symptoms develop. Induction of vomiting with ipecac (C) rarely results in expulsion of the battery. Additionally, vomiting may lead to aspiration and airway compromise and can be especially dangerous if there is esophageal perforation. Activated charcoal should also be avoided because it has not shown benefit, may obscure the view of the endoscopist, and can cause mediastinitis if esophageal perforation is present. Blind removal (A) of batteries with a rubber catheter has been reported, but this does not allow for direct visualization of the esophageal mucosa, which provides important prognostic information. Patients with no evidence of injury on endoscopy can be discharged, but those with significant ulceration or necrosis require admission for management of delayed complications and nutritional support. Blind catheter insertion is also very dangerous in the setting of possible esophageal perforation. Laxatives (D) provide limited benefit in the setting of battery ingestion. If a battery is in the esophagus, it requires immediate removal. If the battery reaches the stomach, it will usually pass on its own without complications. Laxatives can cause diarrhea, abdominal pain, and dehydration and because of the lack of benefit and potential adverse effects, should be avoided. As stated previously, a battery in the esophagus can cause severe caustic injury in as little as two hours with significant delayed morbidity and mortality, and should be removed. It is incorrect to wait (E) for symptoms to develop because tissue injury would have already occurred. If the battery was visualized in the stomach or intestines, then observation would be appropriate.

A 63-year-old man with a history of a cholecystectomy and appendectomy presents with abdominal cramping, vomiting and decreased bowel movements. Bowel sounds are decreased. Which of the following is true regarding this patient? Abdominal X-ray can be used to rule out the diagnosis of small bowel obstruction CT scan of the abdomen is highly specific in small bowel obstruction Serum lactate is highly sensitive early on in patients with small bowel obstruction Serum white blood count is always elevated in small bowel obstruction

Correct Answer ( B ) Explanation: CT scan of the abdomen has rapidly become the imaging modality of choice in diagnosing small bowel obstruction (SBO) as it is both highly sensitive and highly specific. Patients with SBO present with crampy abdominal pain with nausea, vomiting, constipation and abdominal distension. The pain can be generalized and often will come in waves. The rapidity of onset of symptoms depends on how proximal the obstruction is. More proximal obstructions will have a more rapid onset and peak of symptoms. CT scan of the abdomen and pelvis is the diagnostic modality of choice as it has a high sensitivity and specificity and can give additional information including location of obstruction, cause of obstruction and alternative diagnoses.

An 18-year-old man presents with left testicular pain. The pain began two days ago and has gradually gotten worse. He reports a new sexual partner. He does not recall any trauma and reports normal urination. On exam, he is tender to palpation of his posterior left testicle. He has bilateral cremasteric reflexes. No mass is noted on the testes or in the scrotum. What is the most appropriate next step? Order a testicular ultrasound Treatment with ceftriaxone and doxycycline Treatment with ciprofloxacin Urology consultation

Correct Answer ( B ) Explanation: Epididymitis is inflammation of the epididymis most commonly due to bacterial infection. It tends to have a gradual onset of testicular pain with a tender epididymis and an otherwise normal exam. The microbiology of epididymitis depends on the age of the patient. Young boys and men > 35 years of age tend to have epididymitis due to urinary tract infection. Men < 35 years tend to have epididymitis due to sexually transmitted infections. In this 18-year-old sexually active man, the most likely etiology is a sexually transmitted infection (e.g., gonorrhea or chlamydia), and he should be treated with ceftriaxone and doxycycline. A testicular ultrasound (A) is an appropriate diagnostic test if there is uncertainty about the cause of testicular pain. This patient has classic findings of epididymitis. If uncertainty persists, an ultrasound is the diagnostic test of choice. If the patient were older, a urinary tract infection would be the likely cause of his epididymitis and treatment with ciprofloxacin (C) would be the appropriate. A urology consultation (D) should be obtained if testicular torsion is suspected. This is unlikely to be torsion given the time course and physical exam findings

A 49-year-old man with a history of alcoholism presents to the ED for hematemesis. VS are BP 85/40, HR 124, RR 22, and T 36.16°C. Your exam reveals scleral icterus, spider angiomata, and ascites. As the patient is prepped for the operating room, which of the following should be administered in the ED? Norepinephrine Octreotide Propranolol Vitamin K

Correct Answer ( B ) Explanation: Esophageal varices develop in patients with chronic liver disease in response to portal hypertension. Approximately 60% of patients with portal hypertension will develop varices. Of those who develop varices, approximately 25% will experience hemorrhage. Patients who develop varices from alcohol abuse have an even higher risk of bleeding, especially with ongoing alcohol consumption. The patient in the above clinical scenario has evidence of chronic liver disease (scleral icterus, spider angiomata, ascites). Bleeding from esophageal varices is a medical emergency that has a mortality that approaches 25%. Bleeding is often massive. A particular feature of variceal bleeding is the frequency of very early rebleeding, which is a prognostic factor for death. Emergency treatment follows the same principles as for other types of gastrointestinal bleeding and consists of resuscitation, diagnosis, and control of bleeding. Octreotide is a synthetic analogue of somatostatin. It indirectly causes splanchnic vasoconstriction, which leads to decreased portal blood flow and thus decreased bleeding and risk of rebleeding Vitamin K (D) is administered to patients who are bleeding and have a supratherapeutic INR. Vitamin K has no role in the management of acute hemorrhage because it takes at least 4 hours to begin to work. Vitamin K is less efficacious in patients with chronic liver disease because they have a poor capacity to produce the vitamin K-dependent clotting factors. Propranolol (C) is used in cirrhotics as primary prophylaxis to reduce the first occurrence of variceal bleeding. It does not have a role in the management of acute bleeding. Norepinephrine (A) is a vasopressor agent that has alpha- and beta-adrenergic effects. Patients with GI bleeding who are hypotensive are so because they are volume depleted and require saline infusion or blood transfusion rather than vasopressors.

What is the most common cause of acute pancreatitis worldwide? Alcohol Gallstones Medications Trauma

Correct Answer ( B ) Explanation: Gallstones cause about 45% of all acute pancreatitis. Pancreatitis involves edema, inflammation and parenchymal cell death. Local inflammation typically results from obstruction of the pancreatic duct or direct toxicity to pancreatic cells. Inflammation leads to pancreatic enzyme activation either in the pancreatic ducts or within the acinar cells themselves. This premature activation leads to cellular breakdown, focal injury and edema. Gallstones are implicated in roughly 45% of cases and typically results from either a stone becoming lodged in the common bile duct and obstructing outflow from the pancreatic ducts or a stone within the bile duct applies transmural pressure on the pancreatic duct. Either way, obstruction occurs leading to the above pathophysiologic response. Patients typically present with midepigastric pain as well as nausea and vomiting. Treatment focuses on fluid resuscitation, pain control and identification and management of complications (pleural effusions, coagulopathy, shock etc.).

52-year-old man with a history of cirrhosis presents with worsening confusion. His wife reports he has been compliant with his lactulose at home. Which of the following is a common cause of worsening hepatic encephalopathy? Cardiac ischemia Constipation Hyperkalemia Pancreatitis

Correct Answer ( B ) Explanation: Hepatic encephalopathy is a state of confusion caused by acute and chronic liver disease. The pathophysiology involves many pathways but is directly related to the impaired liver's inability to perform its regular metabolic function. Ammonia is used as a marker of the process. Ammonia that is absorbed in the GI tract is metabolized to urea by the liver for excretion. When this does not occur, ammonia levels increase and ultimately cross the blood-brain barrier leading to cerebral dysfunction. Hepatic encephalopathy covers a spectrum of disease from mild cognitive impairment to coma. On physical examination, asterixis is characteristic of the process ("flapping" of the wrist when it is extended). There are many causes of acutely worsening hepatic encephalopathy including constipation due to an increase in intestinal ammonia production and subsequent absorption. Other causes include renal failure, gastrointestinal bleeding, infection, medications with central nervous system activity, and diuretic therapy leading to hypokalemia and alkalosis Cardiac ischemia (A) is not a common cause of hepatic encephalopathy. It is, however, important to think of ischemia in the differential of patients in diabetic ketoacidosis. Hyperkalemia (C) is not typically linked with worsening encephalopathy but hypokalemia is. With decreased serum potassium and the alkalosis that may be associated with this, there is increased conversion of NH4+ to NH3 leading to increased serum levels of ammonia. Pancreatitis (D) alone is not an independent cause of hepatic encephalopathy but in severe cases may lead to dehydration and electrolyte or pH disturbances which can exacerbate the condition.

A mother brings her 11-month-old infant into a rural ED for inconsolable crying. You note the infant lying in the stretcher crying with his knees drawn to his chest. You perform a physical exam and leave the room to order blood work. When you return to the examination room, you note the infant now appears lethargic. An abdominal radiograph is obtained and interpreted by the radiologist as nonspecific. Which of the following diagnostic tests should be performed next? AAbdominal CT scanYour Answer BAbdominal ultrasound CNasogastric tube lavage DUpper GI series

Correct Answer ( B ) Explanation: In patients with suspected intussusception, ultrasound is a quick, noninvasive diagnostic modality commonly used for the diagnosis of intussusception. One study reported that the overall sensitivity and specificity of ultrasonography for detecting ileocolic intussusception was 97.9% and 97.8%, respectively. The authors concluded that ultrasonography should be used as a first-line examination for the assessment of possible pediatric intussusception. Ultrasonography eliminates the risk of exposure to ionizing radiation and can help to depict lead points and residual intussusceptions. It also helps to rule out other possible causes of abdominal pain. On the transverse ultrasound scan, the intussusception appears as a multilayered or wrapped complex mass; longitudinally, it appears as a tube within a tube. Abdominal CT scan (A) is rarely needed to diagnose intussusception and is associated with unnecessary radiation exposure. Nasogastric tube lavage (C) has no role in the management of intussusception. Some patients with intussusception, however, will require nasogastric tube decompression if there is evidence of significant gaseous distention. Upper GI series (D) is used in the diagnosis of pyloric stenosis secondary to delayed emptying from the stomach to the duodenum, due to hypertrophy of the muscle surrounding the pylorus.

Which of the following is true regarding intestinal intussusception? Adults are more affected than children Most adult intussusception cases involve the small bowel Most children with intussusception have a pathologic lesion The classic triad of abdominal pain, mass and heme-positive stools is usually seen in adults with intussusception

Correct Answer ( B ) Explanation: Intussusception in adults is rare but when present, involves the small bowel in 80% of cases. Intussusception involves telescoping of a proximal segment of the bowel into a distal segment. This process causes the mesenteric blood supply to the section of bowel to become compromised leading to ischemia, edema, infarction and possibly gangrene. In adults, intussusception typically presents with symptoms consistent with partial bowel obstruction. Vomiting, bleeding per rectum and constipation are common. The abdomen may become distended as well. As the disease progresses and bowel is compromised, sepsis may ensue. In adults, large bowel obstruction is frequently associated with malignancy. Although plain radiographs can be suggestive of the diagnosis, CT is the definitive study.

A 59-year-old man presents to the ED with 12 hours of emesis and abdominal pain. Vital signs are T 38.2°C, BP 110/79, and HR 109. On exam, you note a tender 2 x 2 cm bulge with erythema in the abdominal midline above the umbilicus. There is abdominal distension, and an occasional high-pitched bowel sound is heard. After placing an IV line and nasogastric tube, which of the following is the most appropriate course of management? AAdminister broad-spectrum antibiotics and attempt reduction BAdminister broad-spectrum antibiotics and emergent surgical consultation CAdminister broad-spectrum antibiotics and obtain a plain radiograph DAdminister broad-spectrum antibiotics then obtain a CT scan of abdomen

Correct Answer ( B ) Explanation: The clinical scenario is consistent with a strangulated loop of bowel (emesis, abdominal pain, fever, erythema, abdominal distension, high-pitched bowel sounds) incarcerated in a ventral hernia. If the contents of a hernia can be returned to their natural cavity by manual reduction, the hernia is termed reducible; if they cannot, it is termed irreducible or incarcerated. Incarcerated hernias (third most common cause of SBO) are subject to inflammatory and edematous changes and are at risk for strangulation, which refers to vascular compromise of the incarcerated contents. When strangulation is not emergently relieved, necrosis and gangrene develop. The treatment for an incarcerated hernia that cannot be manually reduced is surgical fixation. If strangulation is suspected or shock is present, broad-spectrum antibiotics and fluid resuscitation are necessary, but manual reduction is contraindicated because the patient requires emergent surgical management. The reintroduction of ischemic, necrotic bowel back into the peritoneal cavity can result in subsequent perforation and sepsis. If the hernia is incarcerated, but the patient does not yet show signs of strangulation, then an attempt at reduction should be made in the ED.

A 34-year-old man presents for evaluation of abdominal pain. On examination, he is diffusely tender to palpation. His X-ray is shown above. What is the most appropriate management? CT scan of the abdomen and pelvis Exploratory laparotomy Upper endoscopy Upper gastrointestinal series

Correct Answer ( B ) Explanation: The incidence of any perforated viscous increases with age. Patients often have a history of peptic ulcer disease or diverticulitis. In this patient, his age suggests likely peptic ulcer disease as the etiology of his perforation. Patients describe the acute onset of severe epigastric pain that may radiate to the back or shoulders. Approximately 50% of patients experience vomiting. Over time with spillage of contents, peritonitis develops and patients will develop sepsis and shock. In patients with perforated ulcers, abdominal radiography demonstrates free air in 70-80% of cases. With high clinical suspicion, CT Scan should be performed if the patient is not having surgical exploration. In the patient described above with free air present on plain X-ray, an exploratory laparotomy is indicated. Patients need broad-spectrum antibiotics, intravenous hydration and analgesia.

A 24-year-old man presents to the ED with right lower quadrant abdominal pain, nausea, and decreased appetite for two days. His vital signs include a blood pressure of 116/84 mm Hg, heart rate of 104 beats per minute, respiratory rate of 16 breaths per minute, and a temperature of 38.6°C. On examination, the patient has McBurney's point tenderness with associated rebound tenderness. A surgeon is consulted. Which of the following is the most appropriate antibiotic coverage for this patient's condition? Ampicillin Cefoxitin Cephalexin and vancomycin Metronidazole

Correct Answer ( B ) Explanation: The patient has acute appendicitis. When the decision is made to operate, prophylactic antibiotic coverage for gram-negative and anaerobic bacteria should be provided. Second-generation cephalosporins such as cefotetan and cefoxitin provide the appropriate coverage. Ampicillin (A) alone is not adequate due to its poor coverage of anaerobic bacteria. Cephalexin (C) is a first-generation cephalosporin and does not provide adequate gram-negative or anaerobic coverage. The addition of vancomycin extends gram-positive coverage and MRSA coverage. Metronidazole (D) provides adequate anaerobic coverage but poorly covers gram-negative organisms.

A 2-year-old female presents after a witnessed seizure. The parents state that she has not been to day care in 2 days as she has bloody diarrhea and a fever. Her vitals on presentation are T 103.1°F, HR 167, BP 73/43, RR 48, and oxygen saturation 96%. Currently, the patient has a normal neurologic exam and minimal abdominal tenderness. There are no rashes. Labs show a WBC of 19,000 and urinalysis reveals an elevated specific gravity. Which of the following is the most appropriate next step in management? Acetaminophen for fever and follow-up with primary care provider Blood and stool cultures, IV antibiotics, and admission Neurology consultation and admission Oral antibiotics and follow-up with primary care provider

Correct Answer ( B ) Explanation: The patient is suffering from shigellosis. Shigella infection is characterized by high fever, abdominal cramps, diarrhea with mucous and blood, and, in infants, can cause seizures. Although supportive care is the focus of treatment, there are certain populations who should have antibiotic therapy. Although treatment can be parenteral or oral, this patient is young and has high fever, making bacteremia likely. Treatment for children <18 years of age requiring parenteral therapy should be with ceftriaxone 50 mg/kg IV in a single daily dose (maximum 1.5 grams) for 5 days. In children between 6 months and 6 years of age, a simple febrile seizure with return to baseline mental status can be treated with antipyretics and primary care follow-up (A). But this patient does not have a simple febrile seizure. Neurology consultation (C) is not critical for this patient because seizure is commonly seen in infants with shigellosis. Given her fever and likelihood for bacteremia, intravenous, rather than oral (D), antibiotics should be administered.

A 3-week-old boy presents with non-bilious, forceful emesis for two days. On examination, the infant appears active and feeds vigorously followed immediately by vomiting. Which of the following is the most appropriate diagnostic study? Abdominal radiograph Abdominal ultrasound CT scan of the abdomen and pelvis Urine dip

Correct Answer ( B ) Explanation: The patient presents with symptoms consistent with hypertrophic pyloric stenosis, which is most accurately diagnosed by ultrasound of the abdomen. Hypertrophy or hyperplasia of the pyloric sphincter is not present at birth but develops progressively over time. It typically presents in infants between the age of 2 weeks and 2 months. Pyloric stenosis is characterized by projectile, non-bilious vomiting that occurs immediately after feeding due to gastric outlet obstruction. Infants will appear very hungry in between feedings. On examination, peristaltic waves may be seen moving from left to right and the clinician may palpate an "olive" shaped mass in the right upper abdomen. Labs may reveal a hypochloremic metabolic alkalosis in patients with delayed presentations. The cornerstones of diagnosis are either ultrasound or upper GI series. Both have an accuracy of 95%. CT scan of the abdomen and pelvis (C) has never been studied in the diagnosis of pyloric stenosis. It should not be the modality used when there are other options with lower (or no) radiation exposure. In advanced stages of complete pyloric obstruction, plain abdominal radiographs (A) may reveal an enlargement of the stomach and pylorus but sensitivity is low. Urine dip (D) is the first step in the diagnosis of a wide range of diseases most commonly a cystitis (Urinary tract infection). The analysis includes testing for the presence of proteins, glucose, blood, ketones, nitrites, leukocytes, and PH, but it will not diagnose a pyloric stenosis.

A 62-year-old man with a history of alcohol abuse presents to the ED with abdominal pain, fever, and vomiting. His vital signs are temperature 39.05°C, heart rate 114, blood pressure 110/60 mm Hg, respiratory rate 20, and oxygen saturation 99% on room air. On exam, his abdomen is diffusely tender, with guarding in the epigastrium. Lab work reveals a serum white blood cell count of 13 000/mm3 and a lipase of 2450 U/L. All other lab tests are within normal limits. Which of the following is associated with an increased mortality for this patient's diagnosis? Abdominal wall guarding Age >55 years Fever Lipase >2000 U/L

Correct Answer ( B ) Explanation: This patient is presenting with acute pancreatitis. Ranson's criteria is the most commonly used scoring system to aid in the prognosis of acute pancreatitis. The initial score indicates the degree of localized inflammation on hospital admission and is based on five criteria. There are several important limitations of Ranson's criteria. First, the complete score cannot be calculated until the 48-hour mark, diminishing its utility in the ED setting. Second, the original score was derived and validated in patients with alcohol-induced pancreatitis, limiting applicability when other etiologies (i.e., gallstone or viral) are present. Lastly, individuals with HIV/AIDS may have baseline abnormalities of serum calcium and LDH, leading to questionable accuracy in certain patients.

A 43-year-old woman presents to the ED with abdominal pain. She reports that the pain is epigastric and worse with food. She denies any fevers, chills, vomiting, or diarrhea. She recently visited a gastroenterologist who told her she had a bacterial infection in her stomach, however, she did not follow up or receive treatment. On exam, her abdomen is soft and nontender. Which of the following is the most appropriate treatment regimen for this patient's condition? Amoxicillin 1 g BID, metronidazole 500 mg BID, lansoprazole 30 mg BID for 10-14 days Clarithromycin 500 mg BID, amoxicillin 1 g BID, lansoprazole 30 mg BID for 10-14 days Clarithromycin 500 mg BID, amoxicillin 1 g BID, metronidazole 500 mg BID for 10-14 days Clarithromycin 500 mg BID, amoxicillin 1 g BID, metronidazole 500 mg BID, lansoprazole 30 mg BID for 10-14 days

Correct Answer ( B ) Explanation: This patient is presenting with peptic ulcer disease secondary to Helicobacter pylori infection. It is estimated that 30%-40% of the US population is infected with H. pylori, although many will never develop peptic ulcer disease. It is usually acquired through the fecal-oral route, typically in childhood. It is estimated that the majority of patients with duodenal and gastric ulcers are infected with H. pylori. Its presence causes mucosal inflammation and disrupts the normal defense mechanism of the stomach lining, leading to ulceration. Methods of testing include urea breath test, urea blood test, blood antibody testing, stool antigen testing, and biopsy during endoscopy. Triple therapy with clarithromycin 500 mg BID, amoxicillin 1 g BID (metronidazole 500 mg BID if allergic to penicillin), and a proton pump inhibitor (such as lansoprazole 30 mg BID) for 10-14 days is currently recommended for all patients with confirmed H. pylori infection. Amoxicillin 1 g BID, metronidazole 500 mg BID, lansoprazole 30 mg BID for 10-14 days (A) is incorrect as you need the addition of clarithromycin; clarithromycin 500 mg BID, amoxicillin 1 g BID, metronidazole 500 mg BID for 10-14 days (C) is incorrect as you have not used a PPI; and clarithromycin 500 mg BID, amoxicillin 1 g BID, metronidazole 500 mg BID, lansoprazole 30 mg BID for 10-14 days (D) is incorrect as you have treated the patient with too many medications.

A 57-year-old man with a history of chronic kidney disease (baseline Cr = 3.3) and liver cirrhosis presents with confusion. Examination reveals scleral icterus and asterixis. Vital signs and serum glucose are normal. What management is indicated? Head CT and lumbar puncture Lactulose Neomycin Protein restriction diet

Correct Answer ( B ) Explanation: This patient presents with hepatic encephalopathy; a state of disordered cerebral function resulting from acute or chronic liver disease. Encephalopathy in these patients is complex and is related to the liver's inability to perform its normal metabolic functions. Ammonia is a marker of this process. Ammonia is formed primarily in the gastrointestinal tract by the action of bacteria on proteins. Ammonia subsequently crosses the blood-brain barrier and leads to increased glutamine levels in the CNS. There are four stages of hepatic encephalopathy. Diagnosis can be aided with laboratory tests. Ammonia is generally elevated but may not correlate with severity of disease. Liver synthetic function (Prothrombin time, albumin) is usually abnormal. Treatment begins with aggressive supportive care including airway, breathing and circulation assessment. Hepatic encephalopathy patients are usually hemodynamically stable but have a high risk of developing gastrointestinal bleeding, electrolyte abnormalities (hypokalemia and hyponatremia) and hypoglycemia all of which can contribute to changes in mental status. Lactulose, an osmotic cathartic agent, is a primary therapy of this disorder. Lactulose is a poorly absorbed sugar, which is metabolized to lactic acid by colonic bacteria. The resultant acidification of stool traps ammonia as ammonium leading to decreased serum levels. Because it is an osmotic agent, lactulose can cause electrolyte abnormalities and fluid shifts. Neomycin (C) is a poorly absorbed aminoglycoside antibiotic. Oral administration leads to reduction of colonic bacteria responsible for ammonia production. However, neomycin should not be used in those with renal dysfunction as it may further impair renal function. A protein restricted diet (D) is a lifestyle modification that can lead to lower ammonia levels but is a long-term management option that will not help in the acute presentation. In patients with altered mental status, a head CT and lumbar puncture (A) are useful in ruling out infectious causes and space-occupying lesions (tumor, blood). However, the patient's clinical presentation of confusion in the setting of liver cirrhosis and asterixis makes these diagnoses highly unlikely.

A 53-year-old man with a history of atrial fibrillation and hypertension presents with severe abdominal pain. He states that the pain has been there for 3 days but got more severe today. Over the last 3 days, he has been unable to eat because the pain occurs after eating. Vitals signs are T 99.7F, HR 123, BP 101/66, RR 24. Examination reveals an uncomfortable patient with diffuse mild abdominal tenderness to palpation without rebound or guarding. Stool guaiac is positive and the serum lactate is 4.8 mg/dL. A surgical consultation is requested. Which of the following represents the appropriate management? Obtain abdominal X-rays Obtain CT angiogram of the abdomen and pelvis Obtain CT scan of the abdomen and pelvis without IV contrast Obtain right upper quadrant abdominal ultrasound

Correct Answer ( B ) Explanation: This patient's presentation is highly suggestive of mesenteric ischemia, which is best diagnosed by CT angiogram. Once ischemia has progressed to infarction, mortality climbs to 70% underscoring the need for rapid diagnosis and management. Mesenteric arterial embolism is the most common cause of mesenteric ischemia. Risk factors for arterial embolus include coronary artery disease, valvular heart disease and arrhythmias - particularly atrial fibrillation. The classic presentation of mesenteric ischemia is that of an elderly patient who presents with intermittent abdominal pain that is increased with eating. The pain is out of proportion to examination (meaning that the patient complains of severe pain but the exam reveals mild to moderate tenderness and may lack peritoneal signs). Additionally, patients will often have guaiac positive stool testing especially as the bowel becomes more ischemic. An elevated lactate level is also highly suggestive of mesenteric ischemia and has a high sensitivity. Because of the high morbidity and mortality associated with this disease, early surgical consultation and definitive imaging is central to management. The study of choice is either conventional angiography or CT angiography (more easily accessible). Management focuses on aggressive resuscitation, supportive care and early diagnosis. Abdominal radiographs (A) are useful in eliminating other possible diagnoses including small bowel obstruction and bowel perforation but they are not sensitive in diagnosing either of these diseases or mesenteric ischemia. Angiography or CT angiogram should not be delayed while obtaining plain radiographs. Traditional CT scan of the abdomen and pelvis either with or without contrast (C) may show signs of ischemia like bowel wall edema or intramural gas but will not show where the vascular obstruction lies and is not sensitive for detecting mesenteric ischemia. A right upper quadrant ultrasound (D) is the diagnostic modality of choice for biliary disease (cholelithiasis, cholecystitis etc.) but has not been shown to be useful in diagnosing mesenteric ischemia.

A 42-year-old man presents to the Emergency Department with nausea, vomiting, and right upper quadrant abdominal pain. He drinks alcohol daily. Which of the following laboratory results would be most consistent with alcoholic hepatitis? Alkaline phosphatase 350 U/L Aspartate transaminase 1000 U/L and alanine transaminase 1200 U/L Aspartate transaminase 250 U/L and alanine transaminase 120 U/L Mean corpuscular volume 60 fL

Correct Answer ( C ) Explanation: Acute hepatitis can be the result of an infectious process (most commonly viral), toxic injury, or alcohol. Alcoholic hepatitis can range from subclinical disease to acute liver failure. Patients present with nausea, vomiting, and RUQ abdominal pain. On examination, they frequently have a tender, enlarged liver, and possibly jaundice. Laboratory studies include a macrocytic anemia and thrombocytopenia. The WBC count is often elevated as is the prothrombin time and bilirubin. Liver transaminases, alanine transaminase (ALT) and aspartate transaminase (AST) are typically elevated 2-10 times normal. Unlike hepatitis due to other causes, AST is predominantly elevated, often with a AST:ALT ratio of 2:1. While patients with alcoholic hepatitis may have elevations of alkaline phosphatase, levels over three to four times normal (alkaline phosphatase 350 U/L) (A) are more typically seen with obstructive etiologies. Serum transaminases over ten times normal (aspartate transaminase 1000 U/L and alanine transaminase 1200 U/L) (B) are unusual in alcoholic hepatitis and would point to a toxic or viral etiology. A mean corpuscular volume of 60 mL (D) would indicate a microcytic anemia. Chronic alcoholics will have a macrocytic anemia

A 33-year-old man presents to the ED with complaints of abdominal pain and constipation. Physical examination reveals a distended abdomen. An abdominal radiograph shows paucity of gas in the distal colon, mild distension of the small bowel, and marked distension of the large bowel that appears to be bent over on itself with absence of left-sided gas shadows. Which of the following is true regarding this patient's diagnosis? Associated with a fecalith Inflammation involves all layers of the bowel Results from incomplete embryologic fixation of bowel Sigmoidscope detorsion is indicated

Correct Answer ( C ) Explanation: Cecal volvulus results from incomplete embryologic fixation of the cecum, ascending colon, and terminal ileum to the posterior abdominal wall. Although cecal volvulus may be present at any age, the peak incidence occurs in the 20s and 30s. Both sigmoid and cecal volvulus have similar clinical presentations, with abdominal pain (sudden onset, crampy), distension, and constipation. The abdominal plain film usually demonstrates a dilated single loop of bowel in the mid or upper abdomen. It is typically accompanied by distended loops of small bowel. Management is surgical. Appendicitis (A) is associated with a fecalith. Crohn's disease is associated with inflammation of all layers of the bowel (B). Bowel obstruction is a complication of Crohn's disease. Sigmoidoscope detorsion (D) is the initial treatment for a sigmoid volvulus. Due to the proximal nature of the cecum, this cannot be performed for a cecal volvulus; detorsion is done surgically. After detorsion, the cecum is fixed to the abdominal wall, and recurrence is rare.

An 82-year-old nursing-home resident is sent to the emergency department with lower abdominal pain and bloody diarrhea. He has a history of vascular dementia, hypertension, and hyperlipidemia. On examination he is afebrile, and a nasogastric aspirate is negative for evidence of bleeding. Which of the following is the most likely cause of this patient's bleeding? Angiodysplasia Diverticular bleeding Ischemic colitis Peptic ulcer disease

Correct Answer ( C ) Explanation: Ischemic colitis is a syndrome caused by inadequate blood flow through the mesenteric vessels, resulting in ischemia and possible gangrene of the bowel wall. Patients typically present with fairly acute onset crampy abdominal pain with tenderness over the affected bowel. Patients may have bloody diarrhea or passage of frank blood although it is not usually enough to warrant transfusion. The presentation with ischemic colitis differs from acute mesenteric ischemia which presents as pain that is disproportionate to physical examination findings. Risk factors include a history of atherosclerotic disease at other sites, such as coronary artery disease or cerebrovascular disease, advanced age, sepsis and extreme exercise. Bowel wall edema is the most common finding on CT imaging. All cases of ischemic colitis with signs of peritonitis or possible bowel infarction, generally warrant immediate surgical intervention for the resection of the ischemic or necrotic bowel, although this only occurs in about 20% of cases. Most cases resolve with supportive care (eg. IV fluids and bowel rest). Angiodysplasia (A) is a degenerative lesion of previously healthy blood vessels found most commonly in the cecum and results in hematochezia. Bleeding is usually chronic or recurrent and, in most cases, low grade and painless because of the venous source. Diverticular bleeding (B) is the most common cause of lower gastrointestinal hemorrhage. Patients with diverticular bleeding usually present with an abrupt onset of painless rectal hemorrhage. Occasionally, patients may present with mild abdominal cramping or the urge to defecate, secondary to blood within the colon. Peptic ulcer disease (D) can result in an upper gastrointestinal bleed, which often presents with coffee ground emesis, dysphagia, black stools, and chest pain. A negative gastric lavage indicates a very low likelihood of an upper gastrointestinal bleed.

An obese 37-year-old woman is in the emergency room for right-sided abdominal pain and excessive flatulence. This episode has persisted for several hours. On physical exam you palpate her right upper quadrant while she takes a deep breath. The patient experiences pain and has a transient pause in inspiration. This physical exam finding is associated with which of the following signs? Levine sign McBurney's point tenderness Murphy's sign Psoas sign

Correct Answer ( C ) Explanation: Murphy's sign is positive in patients with acute cholecystitis. During inspiration the lungs expand and the diaphragm moves downward. The downward movement pushes the inflamed gallbladder against the examiner's hand which elicits pain. The gallbladder is found in the right upper quadrant just beneath the liver. Acute cholecystitis occurs when a gallstone blocks the cystic duct which prevents the outflow of bile from the gallbladder. This commonly causes pain after eating since bile is used to emulsify fats ingested from a meal. McBurney's point tenderness (B) is found in the right lower quadrant between the umbilicus and the superior iliac spine. This is the general location of where the appendix is connected to the cecum and helps diagnose acute appendicitis. Deep palpation of this area irritates the inflamed appendix eliciting pain. The psoas sign (D) is positive when a patient experiences increased abdominal pain when the right leg is extended at the hip while the patient lies on the left side. This maneuver is used to help diagnose appendicitis. The Levine sign (A) is positive when a patient is holding a clenched fist over their chest to describe dull, pressing chest pain consistent with the discomfort of angina pectoris.

An obese 37-year-old woman presents for right-sided abdominal pain. She normally has the pain after eating, but it usually resolves on its own. This episode has persisted for several hours. On physical exam, you palpate her right upper quadrant while she takes a deep breath. The patient experiences pain and has a transient pause in inspiration. This physical exam finding is associated with which of the following signs? Brudzinski sign Levine sign Murphy's sign Psoas sign

Correct Answer ( C ) Explanation: Murphy's sign is positive in patients with acute cholecystitis. The maneuver is performed by asking the patient to breathe out and then gently placing the hand below the costal margin on the right side at the mid-clavicular line (the approximate location of the gallbladder). The patient is then instructed to inspire. Normally, during inspiration, the abdominal contents are pushed downward as the diaphragm moves down and lungs expand. If the patient stops breathing in (as the gallbladder is tender and, in moving downward, comes in contact with the examiner's fingers) and winces with a 'catch' in breath, the test is considered positive. In order for the test to be considered positive, the same maneuver must not elicit pain when performed on the left side. A sonographic Murphy's sign is positive when pain is elicited when an ultrasound probe is placed over the patient's gallbladder. The Brudzinski sign (A) is positive when flexion of the neck usually causes flexion of the hip and knee. This maneuver is used to help diagnose meningitis. The psoas sign (D) is positive when a patient experiences abdominal pain when they actively flex their leg at the hip and knee. This maneuver is used to help diagnose appendicitis. The Levine sign (B) is positive when a patient is holding a clenched fist over their chest to describe dull, pressing chest pain consistent with the discomfort of angina pectoris.

Which of the following physical exam findings is the most predictive of acute appendicitis? Percussion tenderness Psoas sign Right lower quadrant tenderness Temperature > 38.3°C

Correct Answer ( C ) Explanation: Of all the physical exam findings in patients with suspected appendicitis, right lower quadrant tenderness and abdominal wall rigidity have been shown to have the greatest predictive value. Other findings such as percussion tenderness (A), right lower quadrant pain on thigh extension while lying in left lateral decubitus position (psoas sign) (B), and a temperature > 38.3°C (D) are of lesser utility.

Which of the following is true regarding diverticulitis? All patients should have CT imaging performed Complicated diverticulitis can be treated with oral antibiotics Oral antibiotics should be given for 7-10 days in uncomplicated diverticulitis Ultrasound is the imaging modality of choice

Correct Answer ( C ) Explanation: Patients who present with uncomplicated diverticulitis should be treated with oral antibiotics for 7-10 days. Diverticulitis is an inflammation of the diverticulum in the large intestine. In uncomplicated cases of diverticulitis, patients present with abdominal pain typically in the left lower quadrant with tenderness to palpation in the same area. Patients should not have peritoneal signs or masses on examination. Complicated diverticulitis is defined as the presence of either extensive inflammation or complications such as abscess, peritonitis or obstruction. Patients with uncomplicated diverticulitis can be empirically treated with antibiotics (typically as an outpatient) for 7-10 days. Patients with uncomplicated diverticulitis typically do not require CT imaging (A). A clinical diagnosis of uncomplicated diverticulitis can be made in a patient in the appropriate age range who is exhibiting focal left lower quadrant pain and tenderness in the absence of symptoms or signs that suggest an alternative diagnosis. No mass or peritoneal irritation should be encountered on examination, and the patient should otherwise appear well. If the patient fits this clinical picture, treatment can be initiated on an empirical basis. No laboratory tests or diagnostic imaging is required. Patients with complicated diverticulitis should be treated with intravenous antibiotics (B) and admitted to the hospital. Ultrasound (D) has shown promise in diagnosing diverticulitis but CT is the imaging modality of choice.

A healthy 6-year-old boy presents to the ED with bloody diarrhea. He was in his usual state of health until one week ago when loose, watery stools (up to 10 per day) were noted. He was seen by his pediatrician four days ago but has since developed increasing amounts of blood and pus in his stools along with a low-grade fever. Mom states there is no recent travel, antibiotic use, or known sick contacts. His vitals are heart rate 118 beats per minute, oxygen saturation 100% on room air, and rectal temperature of 38.3°C. Your physical exam reveals a mildly tender abdomen without localization, rebound, guarding, or peritoneal signs. You note grossly bloody stool on rectal exam. A brief discussion with his pediatrician confirms your suspicion of an invasive bacterial diarrhea; a stool culture was positive for Shigella. Which of the following statements is true regarding this condition? Antibiotics should be avoided because this is a severe case and the patient is at highest risk of developing hemolytic uremic syndrome Antidiarrheal agents (such as diphenoxylate and atropine) are indicated, given the frequency of loose stools Extraintestinal manifestations such as hallucinations, confusion, and seizures may occur Oral rehydration should be avoided; IV fluids should be initiated

Correct Answer ( C ) Explanation: Shigella species cause an invasive diarrhea that rarely infects infants younger than three months old and is most common between two and three years of age. Infection is typically transmitted by person-to-person (fecal-oral) contact or through ingestion of contaminated material. Clinical illness varies from mild to severe, with some patients exhibiting abdominal cramps and tenderness. Dysentery (diarrhea with significant blood, pus, and mucus) occurs in approximately 33% of patients. Some patients may also develop extraintestinal manifestations such as reactive arthritis, seizures, and hallucinations. Although antibiotics (A) are generally not indicated for the treatment of mild illness, they should be considered in more severe cases. Hemolytic uremic syndrome (HUS) is a potentially fatal complication that develops when antibiotics are administered to children (but not adults) with enterohemorrhagic E. coli (O157.H7). HUS results from the release of Shigella-like toxin by dying E. coli (and less commonly by infection with S. dysenteriae 1). For this reason, it is recommended that empiric antibiotic therapy be held in pediatric patients with infectious diarrhea pending culture results. In this case, the causative organism is already known, and antibiotic therapy can be safely administered. Antidiarrheal agents (B) are not recommended because they can worsen bacterial invasion of the bowel wall and prolong the infection and carrier state. If dehydration is a concern, oral rehydration (D) should be attempted prior to initiating IV fluids because the patient is not vomiting and is able to tolerate oral intake.

Which of the following is the most common visceral artery aneurysm? Hepatic artery aneurysm Inferior mesenteric artery aneurysm Splenic artery aneurysm Superior mesenteric artery aneurysm

Correct Answer ( C ) Explanation: Splenic artery aneurysms are the most common type of visceral arterial aneurysms, accounting for up to 60% of cases. Etiologies include arterial fibrodysplasia, portal hypertension, and increased splenic AV shunting in pregnancy. Clinical presentation is vague with left upper quadrant pain with radiation to the left shoulder or subscapular area. Most of the aneurysms are < 2 cm in diameter; only 2% result in life-threatening rupture. Treatment is surgical resection if the patient is symptomatic. Otherwise, asymptomatic patients can undergo transcatheter embolization. Of those aneurysms that rupture, up to 95% occur in young pregnant women. Hepatic artery aneurysms (A) represents 20% of visceral artery aneurysms and are caused by atherosclerosis, infection, and abdominal trauma. Clinical presentation can mimic cholecystitis. Inferior mesenteric artery aneurysms (B) are uncommon. Superior mesenteric artery aneurysms (D) are the 3rd most common visceral aneurysms. IV drug abusers are at increased risk.

63-year-old man with a history of hepatitis C presents to the ED with complaints of generalized abdominal pain and distension. Vital signs are HR 110 beats per minute, RR 22 per minute, BP 130/67 mm Hg, T 39.8°C, and oxygen saturation 97% on room air. On exam, his abdomen is tender and moderately distended. Diagnostic paracentesis is performed. Which of the following results should prompt treatment with antibiotics? AAscitic fluid pH of 7.35 BWBC of 275 cells/mm3 with 20% lymphocytes CWBC of 370 cells/mm3 with 90% neutrophils DWBC of 500 cells/mm3 with 40% neutrophils

Correct Answer ( C ) Explanation: Spontaneous bacterial peritonitis (SBP) is an acute bacterial infection of ascitic fluid in patients with liver disease. Patients with SBP lack an apparent external or intra-abdominal focus of infection. Although, by definition, ascites must be present for SBP to develop, free peritoneal fluid may not always be clinically apparent. Definitive diagnosis is made by culture of the ascitic fluid, but treatment decisions should be made prior to culture results. An ascitic fluid granulocyte count >500 correlates with positive cultures in more than 90% of cases; however, ED treatment for SBP should be initiated if the neutrophil count is greater than 250. A pH of <7.34 (A) or a pH gradient between arterial blood and ascitic fluid >0.10 is another reliable early indicator of SBP. The presence of 20% lymphocytes with 275 WBCs (B) correlates with 220 neutrophils, and 40% neutrophils with 500 WBCs (D) correlates with 200 neutrophils, both of which fall below the treatment threshold for SBP.

A mother brings her 22-month-old child to the ED after noticing bloody bowel movements. The child has a normal birth history. Mom denies any pain associated with these episodes, but she states that the child has become increasingly pale with decreased energy. His vital signs include a blood pressure of 95/60 mm Hg, heart rate of 140 beats per minute, respiratory rate of 24 breaths per minute, and oxygen saturation of 98% on room air. Abdominal examination is unremarkable, but his stool is guaiac positive. Which of the following is most likely to confirm the diagnosis? AAbdominal ultrasound BAir contrast enema CTechnetium-99m pertechnetate scintiscan DUpper GI series via nasogastric tube

Correct Answer ( C ) Explanation: The patient's presentation is classic for Meckel's diverticulum, the most common congenital malformation of the GI tract, and follows the rule of 2s: the diverticulum is 2 cm wide and 2 inches long and usually located within 2 feet of the ileocecal valve. The condition occurs in 2% of the population, and only 2% of affected patients ever become symptomatic. Many contain heterotropic tissue, the most common of which are gastric mucosa and pancreatic acini. The classic presentation is painless rectal bleeding in a boy younger than five years. Diagnosis is made by radionuclide scanning (technetium-99m pertechnetate scintiscan), also called a Meckel's scan. The test is highly specific. An abdominal ultrasound (A) is used to diagnose pyloric stenosis which presents as nonbilious projectile vomiting within the first weeks of life. An air contrast enema (B) can be both diagnostic and therapeutic in the evaluation of intussusception which classically presents with intermittent abdominal pain and vomiting. An upper GI series (D) may show the classic corkscrew sign (dilation of the duodenum) in patients with intestinal malrotation and midgut volvulus, a disorder that typically presents with bilious vomiting and abdominal pain.

A 85-year-old nursing home patient presents with diffuse abdominal pain and distension, nausea, but no vomiting. The above abdominal radiograph is obtained. What is the management of this patient? Enema Nasogastric tube and bowel rest Sigmoidoscopy Surgical intervention

Correct Answer ( C ) Explanation: The radiograph demonstrates a markedly dilated single loop of colon consistent with a sigmoid volvulus. This is a closed-loop obstruction that results from twisting of a mobile segment of bowel. These occur almost entirely in two populations: (1) elderly, bed-ridden patients with debilitating comorbid disease and (2) patients of any age with profound neurologic or psychiatric illness. Almost all patients have a history of chronic severe constipation. Although spontaneous reduction of a sigmoid volvulus can occur, it is infrequent enough to mandate procedural intervention. Sigmoidoscopy is used to decompress and detorse the bowel. An enema (A) can be used in constipated patients to help expel stool from the colon, but this is not useful in patients with a known volvulus and can delay definitive therapy. Though often used, nasogastric tube and bowel rest (B) are usually inadequate. Decompression with a sigmoidoscope or even surgery may be necessary for definitive treatment. Surgical intervention (D) with resection and fixation is reserved for failed attempts at decompression with sigmoidoscopy and when there is evidence of bowel gangrene.

A 43-year-old woman with a history of HIV (last CD4 count 231, viral load undetectable) presents with a sore throat that has progressively worsened over 1 week. She has been unable to tolerate solids for the last 3 days and has pain with swallowing liquids as well. Her vitals are normal. On exam, you note the findings seen in the image above. What management is indicated at this time? AAcyclovir BClotrimazole troches CFluconazole DPenicillin V potassium

Correct Answer ( C ) Explanation: This patient has esophagitis secondary to infection with Candida albicans. Infectious esophagitis is uncommon in immunocompetent individuals but is fairly common in immunocompromised patients (i.e. HIV/AIDS, transplant patients, chronic corticosteroid use, diabetes mellitus, chronic alcohol abuse etc.). Changes in the mucosal barrier in the esophagus in these patients leads to an increased susceptibility to infection. Candida species (specifically C. albicans) represent the most common infectious etiologies. Infections with Candida species produce characteristics white plaques. These plaques can be scraped off of mucosal surfaces revealing a raw, erythematous and sometimes bleeding base. In HIV, patients with CD4 counts <200 are more susceptible to Candida infection but this fungus can cause infections in all patients regardless of CD4 count. Although topical agents can be considered in immunocompetent patients with mild disease, patients with immunocompromise will usually require systemic medications (fluconazole, ketoconazole or itraconazole). Treatment usually runs for 3-4 weeks. Acyclovir (A) is recommended for the treatment of infectious esophagitis caused by herpes simplex virus (HSV). In HSV pharyngitis, the presenting complaints are similar to those in Candida esophagitis but examination should reveal herpetic vesicles. Clotrimazole troches (B) and nystatin are topical treatments for Candida esophagitis that can be used in immunocompetent patients with mild symptoms. Penicillin V potassium (Pen VK) (D) is the treatment of choice for pharyngitis secondary to Streptococcus pyongenes. S. pyogenes infection presents with unilateral or bilateral white tonsillar exudates.

A 30-year-old man presents to the ED with three weeks of diarrhea. He reports colicky abdominal pain associated with frequent episodes of pale, loose, foul-smelling stools. He returned from a camping trip in New Hampshire one month prior. His vital signs are temperature 37.1°C, heart rate 85, blood pressure 125/80 mm Hg. Which of the following is the most appropriate treatment for this condition? Ciprofloxacin 500 mg daily for seven days Clindamycin 300 mg QID for seven days Metronidazole 250 mg TID for seven days Rifaximin 200 mg TID for seven days

Correct Answer ( C ) Explanation: This patient is presenting with signs and symptoms consistent with giardiasis, the most common cause of parasitic diarrheal infection in the United States. Fecal-oral transmission of Giardia lamblia occurs with the ingestion of cysts in contaminated water, either municipal water supplies or outdoor water sources (streams and rivers). It is rarely transmitted through food. Giardiasis is noninvasive, and infection remains confined to the lumen of the small intestine. Classic symptoms include explosive diarrhea, colicky abdominal pain, and pale, loose, foul-smelling stools. The incubation period is one to three weeks followed by an abrupt onset. Treatment is empiric. The appropriate regimen is metronidazole 250 mg TID for seven days. Ciprofloxacin (A), a medication commonly used to treat bacterial diarrhea, is not effective in treating parasitic diarrheal infections. Clindamycin (B) has not been shown to be effective against Giardia. Rifaximin (D) is a minimally absorbed antibiotic used in the treatment of traveler's diarrhea and hepatic encephalopathy. Rifaximin is not effective for treatment of Giardia and has limited antimicrobial activity against Campylobacter jejuni, Shigella, and Salmonella species.

A 33-year-old man presents with chest pain and fever after vomiting. Vitals are HR 113, BP 143/73, T 99°F and oxygen saturation 95%. A chest X-ray is shown above. What test should be ordered to make a definitive diagnosis? CT scan of the chest Esophagoduodenoscopy Esophagram with water-soluble contrast MRI of the chest

Correct Answer ( C ) Explanation: This patient presents with signs and symptoms as well as an X-ray concerning for esophageal rupture and should have an esophagram performed emergently. Esophageal perforation or Boerhaave's syndrome, is a life-threatening condition resulting from an increase in intraesophageal pressure. This can occur with forceful vomiting, coughing, heavy lifting or after insufflation during endoscopy. The majority of spontaneous ruptures (90%) occur in the distal esophagus while traumatic rupture usually occurs in the proximal and middle thirds. Patients present with a variety of symptoms including neck and chest pain, dysphagia, respiratory distress, fever, odynophagia, vomiting and hoarseness. Mackler's triad of chest pain, vomiting and subcutaneous emphysema is pathognomonic for the disease but is only seen in half of all cases. Perforation of the lower third of the esophagus may be accompanied by development of a pneumo- or hemothorax and pneumomediastinum. Regardless of location, patients are usually ill-appearing on presentation. Chest X-ray is the first modality that should be used in diagnosis and 90% of patients will have radiographic abnormalities. The diagnosis should be confirmed with contrast radiography in the form of an esophagram with either barium sulfate or water-soluble contrast (the type of contrast is controversial). Typically, a water-soluble contrast is used first and if negative but a clinical suspicion remains high then barium contrast should be used.

A 3-week-old boy presents with two days of non-bilious projectile vomiting. Examination reveals a mass in the infant's right upper quadrant. On a barium upper GI series report, the radiologist states a "string sign" is present. Which of the following is this infant at greatest risk of developing? AHyperchloremia BHyperkalemia CHypokalemia DHyponatremia

Correct Answer ( C ) Explanation: This patient presents with the classic description of congenital pyloric stenosis. Hypertrophic pyloric stenosis describes the narrowing of the pylorus due to congenital or functional hypertrophy of the surrounding muscle. It causes severe projectile non-bilious vomiting and occurs most commonly in male infants of age 2-8 weeks. The hypertrophy is classically described as a palpable olive-shaped mass in the middle upper or right upper quadrant. This condition also occurs in adults, but it is due to a sclerosing stenosis secondary to chronic peptic ulceration. Ultrasound is used to confirm the diagnosis. A barium upper GI series may also show a thin stream of contrast emanating from the pylorus ("string sign" or "railroad track sign"). Any infant with significant vomiting is at risk for metabolic alkalosis due to excessive loss of gastric acid. In an attempt to compensate, cells exchange extracellular potassium for intracellular hydrogen ions, and as such, total body potassium gets shuttled intracellularly, extracellular potassium stores become depleted and hypokalemia ensues. Treatment includes fluid replacement and electrolyte correction. Intravenous and oral atropine can also be used, however, surgery is usually the necessary definitive treatment. Excessive loss of gastric hydrochloric acid not only leads to alkalosis, but also hypochloremia, not hyperchloremia (A). Excessive loss of gastric fluid causes a hypovolemic state, which triggers the adrenal glands to oversecrete aldosterone. Hyperaldosteronism triggers the kidneys to become "volume sparing", thereby actively reabsorbing sodium and excreting potassium, hence the hypernatremia (D). Patient's are at risk for hypokalemia, not hyperkalemia (B).

A patient with palpitations presents to the ED. Her rhythm strip is seen above. Which of the following is the most appropriate initial management? AAmiodarone BCardioversion CMagnesium sulfate DTransvenous pacing at 60-80 bpm

Correct Answer ( C ) Explanation: Torsades de pointes is a form of polymorphic ventricular tachycardia. It is characterized by a fluctuating amplitude of the QRS complexes which appear to twist around the isoelectric line. Torsades is associated with prolonged QT syndrome, hypokalemia and hypomagnesemia. It can deteriorate into ventricular fibrillation. Symptoms include palpitations, dizziness, syncope and sudden death. Acute management begins with intravenous magnesium. Amiodarone (A) prolongs the QT interval. As such, it is contraindicated in torsades de pointes. Cardioversion (B) is a last resort treatment of torsades due to the fact that many cases of torsades are self-limiting and improve with administration of magnesium. A transvenous pacer (D) is typically used to treat bradydysrhythmias. It also can be used to overdrive pace the ventricles which may lead to cessation of torsades. However, inserting a transvenous pacer is time consuming and the initial treatment of torsades is intravenous magnesium.

What is the most common complication after incision and drainage of a pilonidal abscess? Cellulitis Levator ani syndrome Proctalgia fugax Recurrence

Correct Answer ( D ) Explanation: A pilonidal abscess occurs in the midline of the sacrococcygeal area in the natal cleft. It is caused by an infected hair follicle with subsequent abscess formation. The purulent material tracks cephalad and drains to the skin through an epithelialized tract. Patients present with a painful, fluctuant area in the presacral skin. Treatment involves incision and drainage of the abscess; a longitudinal incision lateral to the sacral midline should be used. The recurrence rate of pilonidal abscess is upwards of 40%; therefore, the patient should be referred for follicle removal and unroofing of sinus tracts after the acute inflammation subsides (approximately one week). For patients with chronic refractory disease, marsupialization or wide excision may be required. Cellulitis (A) may complicate the abscess but is not usually a complication of incision and drainage. Levator ani syndrome (B) is precipitated by defecation or prolonged periods of sitting and causes a constant, dull pressure in the sacrococcygeal region. Proctalgia fugax (C) is an intensely painful spasm in the rectal area that originates in the levator muscle complex or the sigmoid colon. It is not a complication of incision and drainage.

A 5-year-old boy presents with abdominal pain, grogginess and vomiting. His father states his son's stools have had a jelly-like appearance for the past 36 hours. You appreciate a sausage-shaped mass during abdominal palpation. Which of the following diagnostic tests is the most appropriate during this child's diagnostic evaluation? Computed tomography KUB radiograph Magnetic resonance imaging Ultrasonography

Correct Answer ( D ) Explanation: Intussusception occurs when a proximal portion of bowel telescopes into a more distal portion, typically with the ileum inserting through the ileocecal valve. It is the most common cause of bowel obstruction in infants between the ages of 3 and 12 months. Its peak incidence is between the ages of 5 and 9 months, although it can occur anytime through childhood and even rarely in adults. Symptoms include vomiting, abdominal pain and lethargy. The classic quality of the affected patient's stool is "currant jelly", a reference to the bloody appearance of mucus-like stool and is usually a late finding. A right hypochondrial sausage-shaped mass, with emptiness appreciated in the right lower quadrant (positive Dance sign) may be palpable. If the obstruction is complete, the abdomen could be quite distended. Clinical diagnosis can be supported with positive findings found on a barium enema, however, for pediatric cases, many recommend ultrasonography as the first-line diagnostic test, as the sensitivity and specificity of this imaging modality are reported to be above 95%. Treatment involves supportive care and an air contrast enema. Surgical intervention is indicated if the contrast enema fails to reduce the intussusception or if the intussusception recurs.

Which of the following is the most common form of liver disease in the United States? Hepatitis A Hepatitis B Hepatitis C Nonalcoholic fatty liver disease

Correct Answer ( D ) Explanation: Nonalcoholic fatty liver (NAFL) disease is, more or less, a benign condition that has become increasingly common in the United States and Western Europe as weight gain and obesity have become more common. It is now the most common cause of liver disorder in the United States and other Western industrialized countries. In fatty liver, the liver functions normally and looks normal under the microscope, except for accumulations of fat within cells. NAFL is often detected when imaging tests of the abdomen are obtained for other reasons (e.g. an ultrasound being done to look for gallstones). Liver blood tests are either normal, or slightly increased. The diagnosis may be confirmed with a right upper quadrant ultrasound examination. If the liver blood tests are significantly elevated or if there are other signs of liver disease (e.g. jaundice), then a liver biopsy may be recommended to look for other problems such as nonalcoholic steatohepatitis (NASH). Aside from losing weight, there is no other treatment, though people with liver disease should be vaccinated against hepatitis A and B if they are not already immune. Hepatitis A (A) is spread via the fecal-oral route directly via person-to-person contact or indirectly through the ingestion of contaminated water or food (raw or undercooked shellfish). There is no carrier state, and it does not cause chronic liver disease. Hepatitis B (B) is spread primarily by the percutaneous route but also may be transmitted by intimate contact via bodily fluids. Fulminant hepatic failure develops in 1% of cases. The carrier state is associated with an increased risk for developing hepatocellular carcinoma. Hepatitis C (C) is the most common form of viral hepatitis in the United States. Prior to 1992, most cases of hepatitis C occurred in association with transfusion of blood and blood products. During this time, hepatitis C was referred to as post-transfusion hepatitis. Currently, intravenous and intranasal drug abuse accounts for the increase in cases.

Which of the following is the most common dysrhythmia associated with the diagnosis of pulmonary embolism? Atrial fibrillation AV-nodal reentrant tachycardia Multifocal atrial tachycardia Sinus tachycardia

Correct Answer ( D ) Explanation: The 12-lead ECG is best used in the setting of suspected PE to provide information about alternative diagnoses such as pericarditis or cardiac ischemia. This is because normal or near normal ECGs with sinus tachycardia or nonspecific ST-T wave changes may be seen up to 30% of the time. When PE causes ECG changes, it is usually the result of acute or subacute pulmonary hypertension. The most common dysrhythmia seen in this setting is sinus tachycardia. Other findings of right-heart strain include T wave inversion in the anterior leads (V1-V4), the S1Q3T3 pattern, and incomplete or complete right bundle branch block. Atrial fibrillation (A) can be seen with pulmonary embolism but is less common than sinus tachycardia. Atrioventricular nodal reentry tachycardia (AVNRT) (B) is the most common type of reentrant supraventricular tachycardia. Pulmonary embolism is an unlikely cause of AVNRT Multifocal atrial tachycardia (MAT) (C) is a dysrhythmia that is seen in a variety of clinical disorders. It is more common in the elderly and is associated with COPD and hypoxia. In addition to a heart rate greater than 100 beats per minute, the characteristic ECG feature is variability in P wave morphology, with each unique P wave morphology felt to indicate a different site of atrial origin. Patients with multiple P wave morphologies but a normal heart rate are considered to have a wandering atrial pacemaker because the heart rate does not meet criteria for a tachycardia.

An overweight 29-year-old roofer presents with acute groin swelling after lifting an 80-pound sack of shingles. Examination reveals a minimally tender mass in the right scrotum. There is no mass in the proximal thigh or abdomen. Which of the following types of hernia do you most likely suspect? Direct inguinal Epigastric Femoral Indirect inguinal

Correct Answer ( D ) Explanation: The inguinal canal is formed by the inguinal ligament and the lower abdominal muscles. It is a tube from the abdominal cavity into the scrotum, allowing the testicles a passageway to descend through. It has two openings called the deep (internal) and superficial (external) inguinal rings. Hernia is the general term used to describe the passage of tissue or organ from its cavity of origin into a different body cavity. Hernias are common in the inguinal region. A direct inguinal hernia is passage of the abdominal contents straight through a weakened portion of the abdominal wall, usually occurring medial to the inferior epigastric vessels and superior to the inguinal ligament. An indirect inguinal hernia is the passage of abdominal contents through the internal inguinal ring, through the inguinal canal, and out through the external inguinal ring into the scrotum. Indirect inguinal hernias are more common than the direct type. Some risk factors that predispose a patient to have an inguinal hernia are obesity, heavy lifting, coughing, straining, and chronic lung disease. A direct inguinal hernia (A) occurs medial to the inferior epigastric vessels in the low abdomen. It causes a mass or bulging in the abdominal wall, a sign which is absent in the above patient. Periumbilical and epigastric hernias (B) typically occur through a weakened portion of the anterior abdominal wall, usually in the midline, through the linea alba. This patient has no abdominal bulging typical of an epigastric hernia. A femoral hernia (C) occurs inferior to the inguinal ligament, causing a mass, edema or bulging in the proximal thigh. It is rare and usually occurs in females.

Which of the following is an appropriate therapy for a patient with severe acute pancreatitis? Antibiotics Anticholinergic agents Calcium chloride Crystalloid infusion

Correct Answer ( D ) Explanation: The management of pancreatitis is primarily supportive. All patients with pancreatitis require fluid resuscitation, as volume depletion is common secondary to inadequate oral intake, vomiting and third space losses. Fluids should be replaced with lactated Ringer's solution or normal saline; several liters may be required. There is emerging evidence that resuscitation with lactated Ringer's results may reduce the incidence of SIRS as compared to normal saline. Vitals signs and urine output should be used to judge the adequacy of volume replacement. Anticholinergics (B) have not been demonstrated to be effective for pancreatitis and can make assessment of ongoing treatment difficult. Antibiotics (A) are not indicated for acute pancreatitis unless there is objective evidence of infection, such as fever or increasing WBC. Calcium chloride (C) has no role in the treatment of pancreatitis.

45-year-old man presents to the ED with vomiting that started six hours prior to arrival. The emesis is coffee-ground in character and is associated with a constant burning pain located in the epigastrium. He reports similar pain over the previous few months that wakes him up from sleep at night. The patient has no known medical problems and takes no medications. He is not a smoker but drinks alcohol socially. His blood pressure is 135/85 mm Hg, heart rate is 95 beats per minute, temperature is 36.8°C, respiratory rate is 16 breaths per minute, oxygen saturation is 100% on room air. The physical exam is remarkable only for discomfort in the epigastrium with deep palpation. Laboratory studies are unremarkable. Which of the following is the most likely diagnosis? AAngiodysplasia BEsophageal varices CMallory-Weiss tear DPeptic ulcer disease EZollinger-Ellison syndrome

Correct Answer ( D ) Explanation: The most common etiology for upper GI bleeding in adults is peptic ulcer disease (PUD). Gastric and duodenal ulcers are grouped together as PUD. Pain that wakes the patient in the middle of the night is a classic symptom because acid production peaks at 2 a.m. Gastric ulcers typically cause pain minutes after eating, whereas discomfort from duodenal ulcers develops 2-3 hours later. The principal cause of PUD is a disruption of the protective gastric mucosal barrier, allowing acid and digestive enzymes to contact the underlying exposed mucosa leading to inflammation, ulceration, and bleeding. Risk factors for gastric mucosal barrier disruption include helicobacter pylori (H. pylori) colonization, nonsteroidal anti-inflammatory drug (NSAID) use, and glucocorticoid use. A causal relationship with ethanol has not been proven, but alcohol abuse may worsen symptoms in patients with preexisting PUD. Initial treatment includes administration of histamine-2 blockers, with proton pump inhibitors for those who have refractory symptoms. Ethanol cessation, avoidance of NSAIDs, and referral to primary care for workup of possible H. pylori infection should be included as well. Angiodysplasia (A) is characterized by lesions in the alimentary tract made up of abnormally thin-walled vessels. They are most commonly in the cecum and ascending colon. When active, angiodysplasia leads to lower GI bleeding and associated abdominal pain. Esophageal varices (B) are characterized by engorged esophageal and gastric veins secondary to portal venous hypertension. Portal hypertension is most commonly associated with cirrhosis, often from long-standing alcohol abuse or chronic viral hepatitis. Clinical features supportive of the diagnosis include jaundice, ascites, spider angioma, caput medusa, Dupuytren's contracture, and palmar erythema. Gynecomastia and testicular atrophy can also be found in males with cirrhosis. A Mallory-Weiss tear (C) is an esophageal injury caused by retching or vomiting and is associated with longitudinal tears at the gastroesophageal junction. Zollinger-Ellison syndrome (E) is caused by a gastrin-secreting tumor in the pancreas, which leads to increased acid production and consequent gastrointestinal ulceration. Zollinger-Ellison syndrome is a rare disease usually associated with intense pain and diarrhea.

A 43-year-old woman presents to the ED with abdominal pain. As you palpate the right upper quadrant, you notice that she stops her inspiration for a brief moment. What is the initial diagnostic modality of choice for this disorder? CT scan Plain film radiograph Radioisotope cholescintigraphy (HIDA scan) Ultrasound

Correct Answer ( D ) Explanation: The most common presenting symptom of cholecystitis is pain, usually in the RUQ. Physical exam reveals tenderness in the RUQ epigastric region. Murphy's sign (tenderness and an inspiratory pause elicited by palpation of the RUQ during inspiration) is suggestive of acute cholecystitis. Ultrasound imaging is the most appropriate initial diagnostic modality. The presence of stones, a thickened gallbladder wall, and pericholecystic fluid has a greater than 50% positive predictive value. Whereas visualization of the gallbladder without identification of stones has an extremely high negative predictive value for acute cholecystitis. CT scan (A) is also excellent at identifying acute cholecystitis and has a sensitivity of 92% and specificity of 99%. Given that CT scanning involves ionizing radiation, it is preferred that ultrasound is the initial diagnostic modality. CT scanning may be useful when there are other intra-abdominal disorders in consideration. Plain film radiographs (B) demonstrate stones in the gallbladder less than 25% of the time. Radioisotope cholescintigraphy (HIDA scan) (C) has a higher sensitivity and specificity than ultrasound, making it the most accurate study for cholecystitis; however, it is reserved for cases where US is negative or equivocal.

A 47-year-old man with chronic low back pain presents with epigastric pain for 3 weeks. The pain is burning, without radiation and occurs 2-3 hours after eating. Vital signs are normal. He takes ibuprofen and naproxen for his back pain. What management is indicated? AAdmit for endoscopy BStart esomeprazole and refer for outpatient evaluation CStart ranitidine and refer for outpatient evaluation DStop ibuprofen and naproxen and refer for outpatient evaluation

Correct Answer ( D ) Explanation: The patient presents with clinical gastritis likely secondary to the use of non-steroidal anti-inflammatory drugs (NSAIDs). Technically, gastritis is a histologic diagnosis indicating inflammation of the gastric mucosa and can only be definitively made after endoscopy and biopsy. Clinicians, however, refer to the symptoms of dyspepsia (pain in the upper abdomen that presents as bloating or heartburn) as gastritis. The most common cause of gastritis is Helicobacter pylori infection but it also commonly results from medications including salicylates and NSAIDs (ibuprofen, naproxen etc.). Acute gastritis typically presents with abdominal pain located in the epigastric area, which is burning in nature and can also have bloating or nausea associated with it. Treatment should begin with removal of any possible inciting agents including alcohol, smoking, steroids or NSAIDs. Ranitidine (C) is a histamine H2 receptor antagonist and esomeprazole (B) is a proton-pump inhibitor (PPI). Both of these drugs act by reducing acid production in the stomach and can reduce dyspepsia. However, these drugs should be second line to removing any inciting agents. Endoscopy (A) can be performed to definitively diagnose gastritis and to rule out other causes of symptoms (peptic ulcer disease, gastric tumor etc.). However, endoscopy is not indicated in the emergent setting for mild to moderate symptoms. Referral to a gastroenterologist for consideration of an upper endoscopy is reasonable.

A 73-year-old man presents with vomiting and abdominal pain for 2 days. The patient has a remote history of cholecystectomy and appendectomy. Examination reveals a markedly distended abdomen and absent bowel sounds. Lab studies show an elevated WBC count and a lactate of 4.3 mmol/L. An abdominal radiograph is obtained that is shown above. Which of the following is the most appropriate management at this time? Administer intravenous fluids and obtain a CT of the abdomen and pelvis Arrange for emergent colonoscopy Perform a soapsuds enema and administer polyethylene glycol Place a nasogastric tube, begin antibiotics and obtain a surgical consultation

Correct Answer ( D ) Explanation: This patient presents with a high-grade small bowel obstruction (SBO) with evidence of bowel ischemia (elevated lactate). Mortality has fallen in the last century with aggressive surgical treatment (from 60% to 5%). The abdominal radiograph above shows multiple air-fluid levels consistent with an SBO. Radiographs are abnormal in 50-60% of cases and are more likely to demonstrate abnormality when the obstruction is high-grade versus partial. Two views (upright and supine or supine and decubitus) should be obtained. Mechanical obstruction refers to the presence of a physical barrier to the flow of intestinal contents. In a simple obstruction, the intestinal lumen is partially or completely obstructed causing intestinal distension proximally but does not cause compromise of the vascular supply. In a closed-loop obstruction, a segment of bowel is obstructed at two sequential sites usually by twisting on a hernia opening or adhesive band leading to compromise of blood flow eventually resulting in bowel ischemia. Ischemia may only be seen on CT scan or occasionally, on laparoscopy or laparotomy. However, an elevated lactate in the setting of an SBO is highly suggestive of intestinal ischemia. The presence of blood in stool (either gross blood or guaiac positive stools) also suggests the presence of ischemia or infarction. When compromise of the vascular supply is suspected, the patient should have an emergent surgical consultation for operative management. Immediate management should also include placement of a nasogastric tube for decompression of the proximal parts of the intestines, intravascular volume resuscitation and intravenous antibiotics when vascular compromise is suspected or confirmed.

A 34-year-old man, with no past medical history, presents to the emergency department complaining of burning epigastric pain radiating to the back for the last three months. The pain starts a couple of hours after eating and occurs at night. The patient does not take any medications. Vitals are T 37°C, HR 82, BP 138/62 RR 18, and oxygen saturation 100% on room air. Physical examination is significant for brown, heme-positive stool. Once the diagnosis is confirmed, which of the following is the most appropriate to treat this condition? Antacids Aspirin Carafate Clarithromycin, amoxicillin, and a proton-pump inhibitor (PPI)

Correct Answer ( D ) Explanation: This patient presents with symptoms suggestive of peptic ulcer disease (PUD). PUD is the collective term for gastric and duodenal ulcers. PUD is a common disease affecting 4 million patients in the U.S. each year. The most common etiologic agent of PUD is infection with Helicobacter pylori and eradication of the bacteria leads to more rapid ulcer healing, prevents relapse and decreases the rate of complications. The second most common cause of PUD is the use of non-steroidal anti-inflammatory drugs (NSAIDs). About 1% of patients develop PUD secondary to increased levels of gastrin from gastrin-secreting tumors. Patients typically present with epigastric abdominal pain which occurs 2-5 hours after eating or when sleeping. Pain is often relieved after eating or with antacids. Complications include perforation, upper gastrointestinal hemorrhage and gastric outlet obstruction. Initial treatment of presumptive PUD (definitive diagnosis can only be made on endoscopy) includes lifestyle changes including smoking cessation, reduction or elimination of alcohol use, and stopping NSAID and aspirin use. If NSAIDs are not being used by a patient with suspected PUD (as in this case), the current recommendation is to test for H. pylori with antibody detection, urea breath test or fecal antigen tests. This is often done on an outpatient basis with a gastroenterologist or primary care provider. Once H.pylori is confirmed as the cause, triple therapy, which consists of clarithromycin 500 mg twice a day, amoxicillin 1 gram twice a day (or metronidazole 500 mg twice a day if penicillin allergic) and a PPI once a day for 10-14 days, should be initiated. Aspirin (A) ​will exacerbate ulcers, not treat them. Antacids (B) (e.g. bismuth subsalicylate, magnesium hydroxide) can give symptomatic relief of dyspepsia but do not treat the underlying cause of PUD and may delay definitive treatment by reducing symptoms. Carafate (C) can provide symptomatic care but will not eradicate H. pylori.

A 23-year-old man presents with abdominal pain, vomiting and two loose, nonbloody stools. Physical examination reveals right lower quadrant tenderness to palpation. A CT is performed showing a normal appendix and some inflammation at the ileocecal junction. What pathogen is commonly implicated in this disorder? AAeromonas species BSalmonella enterica CVibrio parahaemolyticus DYersinia enterocolitica

Correct Answer ( D ) Explanation: Yersinia enterocolitica infection can cause an ileocecitis, which may mimic the signs and symptoms of acute appendicitis. Y. enterocolitica infection results from oral ingestion of the bacteria with subsequent invasion of intestinal mucosa. About 2/3 of patients present with features of gastroenteritis - diarrhea, nausea, vomiting, colicky abdominal pain and fever. About 1/3 of patients will present with minimal or no diarrhea and a clinical presentation similar to appendicitis. Y. enterocolitica infection is typically self limited and requires supportive care only. Aeromonas species (A), Salmonella enterica (B), Vibrio parahaemolyticus (C) all cause a typical gastroenteritis picture but do not mimic appendicitis.

One Step Further Question: What is fetor hepaticus?

Answer: A musty, sweet odor caused by the inability of the liver to metabolize mercaptans produced by bacteria.

One Step Further Question: What is the name of the surgery used to correct hypertrophic pyloric stenosis?

Answer: A pyloromyotomy, known as the Ramstedt procedure.

One Step Further Question: Which is more elevated in alcoholic hepatitis, AST or ALT?

Answer: AST>ALT in alcoholic liver disease, compared to ALT>AST in viral hepatitis.

One Step Further Question: When should abscesses associated with diverticulitis be surgically treated?

Answer: Abscesses over 5 cm in diameter should either be drained percutaneously or surgically.

One Step Further Question: What are the three most common causes of SBO in the US?

Answer: Adhesions from prior surgeries, hernias and tumors.

One Step Further Question: What are the components of Ranson's Criteria on admission?

Answer: Age, WBC, glucose, LDH and AST.

One Step Further Question: Which is the gold standard test for diagnosis of visceral artery aneurysms?

Answer: Angiogram.

One Step Further Question: What antibiotics can be used to treat shigellosis in patients with cephalosporin allergy?

Answer: Azithromycin or ciprofloxacin.

One Step Further Question: What is quadruple therapy for peptic ulcer disease?

Answer: Bismuth subsalicylate, metronidazole, tetracycline, and a PPI.

One Step Further Question: What is the most common cause of bowel obstruction in pregnancy?

Answer: Cecal volvulus.

One Step Further Question: What is the most accurate radiographic test to diagnose acute cholecystitis?

Answer: Cholescintigraphy is the most accurate radiographic test for diagnosing acute cholecystitis.

One Step Further Question: What is Dance's sign?

Answer: Considered pathognomonic for intussusception: a sausage-like mass in the RUQ representing the actual intussusceptum and an empty space in the RLQ representing the movement of the cecum out of its normal position.

One Step Further Question: Chronic diarrhea in patients with AIDS is most commonly caused by which pathogen?

Answer: Cryptosporidium.

One Step Further Question: Which 2 populations have a high rate of appendiceal perforation?

Answer: Elderly and small children.

One Step Further Question: In what population is sigmoid volvulus most common?

Answer: Elderly, debilitated, nursing home population.

One Step Further Question: What other gastrointestinal pathogen can give an appendicitis-like presentation?

Answer: Entamoeba histolytica.

One Step Further Question: Which organism is the most common cause of spontaneous bacterial peritonitis?

Answer: Escherichia coli.

One Step Further Question: In patients identified as high risk for spontaneous bacterial peritonitis, which antibiotics are used prophylactically?

Answer: Fluoroquinolones or TMP-SMX.

One Step Further Question: In the setting of liver cirrhosis, why is glucagon unlikely to help with episodes of hypoglycemia?

Answer: Glucagon acts by increasing the breakdown of glycogen to glucose. Patients with liver cirrhosis do not have adequate glycogen stores for glucagon to be effective.

One Step Further Question: What is the most common endocrine complication in chronic pancreatitis?

Answer: Glucose intolerance.

One Step Further Question: What is the most common predisposing factor for peptic ulcer disease in adults?

Answer: H. pylori colonization.

One Step Further Question: Which hepatitis is caused by a defective RNA-containing virus?

Answer: Hepatitis D (delta)

One Step Further Question: What is the name of the anatomic structure through which a direct inguinal hernia usually protrudes?

Answer: Hesselbach's triangle (bordered by the abdominus rectus muscle, inguinal ligament and inferior epigastric vessels).

One Step Further Question: What is the most commonly occurring hernia?

Answer: Indirect inguinal hernia.

One Step Further Question: What type of necrosis can button batteries lead to?

Answer: Liquefaction necrosis.

One Step Further Question: What positional changes can be advised to relieve pain due to epididymitis?

Answer: Lying recumbent with testicular elevation.

One Step Further Question: What important pathology classically presents with bilious emesis in infants?

Answer: Malrotation with volvulus must be suspected with bilious vomiting in infants.

One Step Further Question: Which medications are recommended for the treatment of intussusception?

Answer: Medications are not currently a component in the care of intussusception.

One Step Further Question: Is pilonidal abscess more common in males or females?

Answer: More common in males (4:1 male predominance).

One Step Further Question: What are two common causes of viral pancreatitis?

Answer: Mumps and coxsackie B virus.

One Step Further Question: What thoracic pathologies should always be considered in patients with abdominal pain?

Answer: Myocardial infarction (or ischemia), pneumonia, pulmonary embolism, pericarditis and myocarditis.

One Step Further Question: In addition to lactulose, what is another treatment for hepatic encephalopathy?

Answer: Neomycin, Rifaximin.

One Step Further Question: Is an NG tube required in acute pancreatitis?

Answer: No, it is recommended for patients with intractable vomiting or ileus.

One Step Further Question: Does a positive Murphy's sign always indicate acute cholecystitis?

Answer: No. Patients with severe biliary colic may have a Murphy's sign and RUQ pain for several days after the stone has passed.

One Step Further Question: What is the definition of odynophagia and dysphagia?

Answer: Odynophagia means pain on swallowing, dysphagia means difficulty swallowing

One Step Further Question: What is the "pseudo-kidney sign?"

Answer: On ultrasound, the intussuscepted segment may have an appearance similar to that of a kidney.

One Step Further Question: What is the most common cause of free intraperitoneal air?

Answer: Perforated peptic ulcer.

One Step Further Question: What are two clinical situations that can lead to a false-negative D-dimer?

Answer: Recent anticoagulation and subacute thrombosis (>7 days).

One Step Further Question: Name two congenital QT-prolongation syndromes?

Answer: Romano-Ward syndrome and Jervell-Lange-Nielsen syndrome.

One Step Further Question: What is the most common cause of dysentery in the United States?

Answer: Shigella sonnei. Although there are four Shigella species, S. sonnei is the most common and has only one known subtype. Other species include S. dysenteriae, S. flexneri, and S. boydii. Each has multiple subtypes.

One Step Further Question: What is the recommended antibiotic regimen for perforated appendicitis?

Answer: Single agent therapy is effective. Agents include a second-generation cephalosporin, ampicillin-sulbactam, or meropenem.

One Step Further Question: What are three lifestyle changes that should be recommended to patients with peptic ulcer disease?

Answer: Smoking cessation, alcohol abstinence, and avoidance of NSAIDS (including aspirin).

One Step Further Question: What organisms are typically implicated in anorectal abscesses?

Answer: Staphylococcus aureus, Escherichia coli, Streptococcus, Proteus and Bacteroides.

One Step Further Question: What is the appropriate pediatric dosing for intravenous fluid boluses in hypotensive pediatric patients?

Answer: The dosing is 20 mL/kg.

One Step Further Question: In which portion of the esophagus does rupture typically occur?

Answer: The posterolateral portion is the most common site of rupture.

One Step Further Question: What blood vessel is most commonly affected in mesenteric arterial embolism?

Answer: The superior mesenteric artery (SMA) is the most common site of arterial emoblism.

One Step Further Question: What medications are recommended for treatment of H. pylori associated peptic ulcer disease?

Answer: Triple therapy with clarithromycin, amoxicillin (or metronidazole) and a PPI for 10-14 days.

One Step Further Question: What is the most common cause of intussusception in adults?

Answer: Unlike children where most intussusception is idiopathic, 65% of adult cases are due to tumors (either benign or malignant).


Related study sets

Business Law: Chapter 1- Legal and Constitutional Foundations of Business

View Set

Computer Software & Hardware Architecture CIS

View Set

Healthcare Management Student Made Quizzes

View Set

AP Biology: Chapter 18: Regulation of Gene Expression

View Set

Chapter 5 - Adaptations to Anaerobic Training Programs

View Set

Medical Terminology Chapter 2: Congenital diseases

View Set

Place Value/ Standard and Expanded Form

View Set